Let $a, b, c gt{1}$ be integers such that $gcd(a − 1, b − 1, c − 1) gt{1}$ Prove that $abc − 1$ is...











up vote
1
down vote

favorite













Let $a, b, c gt{1}$ be integers such that gcd$(a − 1, b − 1, c − 1) gt{1}$ Prove that $abc − 1$ is not a prime




I have been trying to tackle this question for some time and I got stuck multiple. So far I denoted that gcd$(a − 1, b − 1, c − 1)=d$ $$therefore a-1 equiv 0 pmod d, b-1 equiv 0 pmod d, c-1 equiv 0 pmod d$$ $$therefore a equiv 1 pmod d, b equiv 1 pmod d, c equiv 1 pmod d$$ $$therefore abc equiv 1 pmod d iff abc-1 equiv 0 pmod d$$ However, I realized that $abc-1$ can still be prime if $d$ was prime and $abc-1=d$ So I attempted to assume contradiction and that gcd$(a − 1, b − 1, c − 1)=abc-1$ but failed to acheive anything. Is there a way I can carry on from here. Thank you anyways.










share|cite|improve this question


















  • 1




    As $a,b,cge d+1$ so, $abc-1ge (d+1)^3-1=d^3+3d^2+3d$
    – Offlaw
    Nov 24 at 7:51






  • 1




    $abc - 1 = (a-1)bc + (b-1)c + (c-1)$
    – achille hui
    Nov 24 at 7:53















up vote
1
down vote

favorite













Let $a, b, c gt{1}$ be integers such that gcd$(a − 1, b − 1, c − 1) gt{1}$ Prove that $abc − 1$ is not a prime




I have been trying to tackle this question for some time and I got stuck multiple. So far I denoted that gcd$(a − 1, b − 1, c − 1)=d$ $$therefore a-1 equiv 0 pmod d, b-1 equiv 0 pmod d, c-1 equiv 0 pmod d$$ $$therefore a equiv 1 pmod d, b equiv 1 pmod d, c equiv 1 pmod d$$ $$therefore abc equiv 1 pmod d iff abc-1 equiv 0 pmod d$$ However, I realized that $abc-1$ can still be prime if $d$ was prime and $abc-1=d$ So I attempted to assume contradiction and that gcd$(a − 1, b − 1, c − 1)=abc-1$ but failed to acheive anything. Is there a way I can carry on from here. Thank you anyways.










share|cite|improve this question


















  • 1




    As $a,b,cge d+1$ so, $abc-1ge (d+1)^3-1=d^3+3d^2+3d$
    – Offlaw
    Nov 24 at 7:51






  • 1




    $abc - 1 = (a-1)bc + (b-1)c + (c-1)$
    – achille hui
    Nov 24 at 7:53













up vote
1
down vote

favorite









up vote
1
down vote

favorite












Let $a, b, c gt{1}$ be integers such that gcd$(a − 1, b − 1, c − 1) gt{1}$ Prove that $abc − 1$ is not a prime




I have been trying to tackle this question for some time and I got stuck multiple. So far I denoted that gcd$(a − 1, b − 1, c − 1)=d$ $$therefore a-1 equiv 0 pmod d, b-1 equiv 0 pmod d, c-1 equiv 0 pmod d$$ $$therefore a equiv 1 pmod d, b equiv 1 pmod d, c equiv 1 pmod d$$ $$therefore abc equiv 1 pmod d iff abc-1 equiv 0 pmod d$$ However, I realized that $abc-1$ can still be prime if $d$ was prime and $abc-1=d$ So I attempted to assume contradiction and that gcd$(a − 1, b − 1, c − 1)=abc-1$ but failed to acheive anything. Is there a way I can carry on from here. Thank you anyways.










share|cite|improve this question














Let $a, b, c gt{1}$ be integers such that gcd$(a − 1, b − 1, c − 1) gt{1}$ Prove that $abc − 1$ is not a prime




I have been trying to tackle this question for some time and I got stuck multiple. So far I denoted that gcd$(a − 1, b − 1, c − 1)=d$ $$therefore a-1 equiv 0 pmod d, b-1 equiv 0 pmod d, c-1 equiv 0 pmod d$$ $$therefore a equiv 1 pmod d, b equiv 1 pmod d, c equiv 1 pmod d$$ $$therefore abc equiv 1 pmod d iff abc-1 equiv 0 pmod d$$ However, I realized that $abc-1$ can still be prime if $d$ was prime and $abc-1=d$ So I attempted to assume contradiction and that gcd$(a − 1, b − 1, c − 1)=abc-1$ but failed to acheive anything. Is there a way I can carry on from here. Thank you anyways.







elementary-number-theory






share|cite|improve this question













share|cite|improve this question











share|cite|improve this question




share|cite|improve this question










asked Nov 24 at 7:45









user587054

31810




31810








  • 1




    As $a,b,cge d+1$ so, $abc-1ge (d+1)^3-1=d^3+3d^2+3d$
    – Offlaw
    Nov 24 at 7:51






  • 1




    $abc - 1 = (a-1)bc + (b-1)c + (c-1)$
    – achille hui
    Nov 24 at 7:53














  • 1




    As $a,b,cge d+1$ so, $abc-1ge (d+1)^3-1=d^3+3d^2+3d$
    – Offlaw
    Nov 24 at 7:51






  • 1




    $abc - 1 = (a-1)bc + (b-1)c + (c-1)$
    – achille hui
    Nov 24 at 7:53








1




1




As $a,b,cge d+1$ so, $abc-1ge (d+1)^3-1=d^3+3d^2+3d$
– Offlaw
Nov 24 at 7:51




As $a,b,cge d+1$ so, $abc-1ge (d+1)^3-1=d^3+3d^2+3d$
– Offlaw
Nov 24 at 7:51




1




1




$abc - 1 = (a-1)bc + (b-1)c + (c-1)$
– achille hui
Nov 24 at 7:53




$abc - 1 = (a-1)bc + (b-1)c + (c-1)$
– achille hui
Nov 24 at 7:53










2 Answers
2






active

oldest

votes

















up vote
1
down vote













Note that:
$$
(md+1)(nd+1)(pd+1)-1=dtimes[m+n+p+d(mn+np+pm)+d^2mnp].
$$

Both factors are bigger than $1$.






share|cite|improve this answer

















  • 1




    since $a,b,c,dgt1$, we know that $(m+n+p)+(mn+np+pm)d+mnpd^2ge13$
    – robjohn
    Nov 24 at 18:37


















up vote
0
down vote













Hint $ d,mid, abc!-!1 $ by $bmod d!: a,b,cequiv 1,Rightarrow, abcequiv 1^3equiv 1$



& $ 1 < d < abc! -! 1 $ by $ dmid a!-!1 < acolor{#c00}{bc}!-!1,$ by $,color{#c00}{bc} > 1$






share|cite|improve this answer























    Your Answer





    StackExchange.ifUsing("editor", function () {
    return StackExchange.using("mathjaxEditing", function () {
    StackExchange.MarkdownEditor.creationCallbacks.add(function (editor, postfix) {
    StackExchange.mathjaxEditing.prepareWmdForMathJax(editor, postfix, [["$", "$"], ["\\(","\\)"]]);
    });
    });
    }, "mathjax-editing");

    StackExchange.ready(function() {
    var channelOptions = {
    tags: "".split(" "),
    id: "69"
    };
    initTagRenderer("".split(" "), "".split(" "), channelOptions);

    StackExchange.using("externalEditor", function() {
    // Have to fire editor after snippets, if snippets enabled
    if (StackExchange.settings.snippets.snippetsEnabled) {
    StackExchange.using("snippets", function() {
    createEditor();
    });
    }
    else {
    createEditor();
    }
    });

    function createEditor() {
    StackExchange.prepareEditor({
    heartbeatType: 'answer',
    convertImagesToLinks: true,
    noModals: true,
    showLowRepImageUploadWarning: true,
    reputationToPostImages: 10,
    bindNavPrevention: true,
    postfix: "",
    imageUploader: {
    brandingHtml: "Powered by u003ca class="icon-imgur-white" href="https://imgur.com/"u003eu003c/au003e",
    contentPolicyHtml: "User contributions licensed under u003ca href="https://creativecommons.org/licenses/by-sa/3.0/"u003ecc by-sa 3.0 with attribution requiredu003c/au003e u003ca href="https://stackoverflow.com/legal/content-policy"u003e(content policy)u003c/au003e",
    allowUrls: true
    },
    noCode: true, onDemand: true,
    discardSelector: ".discard-answer"
    ,immediatelyShowMarkdownHelp:true
    });


    }
    });














    draft saved

    draft discarded


















    StackExchange.ready(
    function () {
    StackExchange.openid.initPostLogin('.new-post-login', 'https%3a%2f%2fmath.stackexchange.com%2fquestions%2f3011293%2flet-a-b-c-gt1-be-integers-such-that-gcda-%25e2%2588%2592-1-b-%25e2%2588%2592-1-c-%25e2%2588%2592-1-gt1-pro%23new-answer', 'question_page');
    }
    );

    Post as a guest















    Required, but never shown

























    2 Answers
    2






    active

    oldest

    votes








    2 Answers
    2






    active

    oldest

    votes









    active

    oldest

    votes






    active

    oldest

    votes








    up vote
    1
    down vote













    Note that:
    $$
    (md+1)(nd+1)(pd+1)-1=dtimes[m+n+p+d(mn+np+pm)+d^2mnp].
    $$

    Both factors are bigger than $1$.






    share|cite|improve this answer

















    • 1




      since $a,b,c,dgt1$, we know that $(m+n+p)+(mn+np+pm)d+mnpd^2ge13$
      – robjohn
      Nov 24 at 18:37















    up vote
    1
    down vote













    Note that:
    $$
    (md+1)(nd+1)(pd+1)-1=dtimes[m+n+p+d(mn+np+pm)+d^2mnp].
    $$

    Both factors are bigger than $1$.






    share|cite|improve this answer

















    • 1




      since $a,b,c,dgt1$, we know that $(m+n+p)+(mn+np+pm)d+mnpd^2ge13$
      – robjohn
      Nov 24 at 18:37













    up vote
    1
    down vote










    up vote
    1
    down vote









    Note that:
    $$
    (md+1)(nd+1)(pd+1)-1=dtimes[m+n+p+d(mn+np+pm)+d^2mnp].
    $$

    Both factors are bigger than $1$.






    share|cite|improve this answer












    Note that:
    $$
    (md+1)(nd+1)(pd+1)-1=dtimes[m+n+p+d(mn+np+pm)+d^2mnp].
    $$

    Both factors are bigger than $1$.







    share|cite|improve this answer












    share|cite|improve this answer



    share|cite|improve this answer










    answered Nov 24 at 7:50









    yurnero

    7,3701925




    7,3701925








    • 1




      since $a,b,c,dgt1$, we know that $(m+n+p)+(mn+np+pm)d+mnpd^2ge13$
      – robjohn
      Nov 24 at 18:37














    • 1




      since $a,b,c,dgt1$, we know that $(m+n+p)+(mn+np+pm)d+mnpd^2ge13$
      – robjohn
      Nov 24 at 18:37








    1




    1




    since $a,b,c,dgt1$, we know that $(m+n+p)+(mn+np+pm)d+mnpd^2ge13$
    – robjohn
    Nov 24 at 18:37




    since $a,b,c,dgt1$, we know that $(m+n+p)+(mn+np+pm)d+mnpd^2ge13$
    – robjohn
    Nov 24 at 18:37










    up vote
    0
    down vote













    Hint $ d,mid, abc!-!1 $ by $bmod d!: a,b,cequiv 1,Rightarrow, abcequiv 1^3equiv 1$



    & $ 1 < d < abc! -! 1 $ by $ dmid a!-!1 < acolor{#c00}{bc}!-!1,$ by $,color{#c00}{bc} > 1$






    share|cite|improve this answer



























      up vote
      0
      down vote













      Hint $ d,mid, abc!-!1 $ by $bmod d!: a,b,cequiv 1,Rightarrow, abcequiv 1^3equiv 1$



      & $ 1 < d < abc! -! 1 $ by $ dmid a!-!1 < acolor{#c00}{bc}!-!1,$ by $,color{#c00}{bc} > 1$






      share|cite|improve this answer

























        up vote
        0
        down vote










        up vote
        0
        down vote









        Hint $ d,mid, abc!-!1 $ by $bmod d!: a,b,cequiv 1,Rightarrow, abcequiv 1^3equiv 1$



        & $ 1 < d < abc! -! 1 $ by $ dmid a!-!1 < acolor{#c00}{bc}!-!1,$ by $,color{#c00}{bc} > 1$






        share|cite|improve this answer














        Hint $ d,mid, abc!-!1 $ by $bmod d!: a,b,cequiv 1,Rightarrow, abcequiv 1^3equiv 1$



        & $ 1 < d < abc! -! 1 $ by $ dmid a!-!1 < acolor{#c00}{bc}!-!1,$ by $,color{#c00}{bc} > 1$







        share|cite|improve this answer














        share|cite|improve this answer



        share|cite|improve this answer








        edited Nov 24 at 19:10

























        answered Nov 24 at 17:43









        Bill Dubuque

        207k29189624




        207k29189624






























            draft saved

            draft discarded




















































            Thanks for contributing an answer to Mathematics Stack Exchange!


            • Please be sure to answer the question. Provide details and share your research!

            But avoid



            • Asking for help, clarification, or responding to other answers.

            • Making statements based on opinion; back them up with references or personal experience.


            Use MathJax to format equations. MathJax reference.


            To learn more, see our tips on writing great answers.





            Some of your past answers have not been well-received, and you're in danger of being blocked from answering.


            Please pay close attention to the following guidance:


            • Please be sure to answer the question. Provide details and share your research!

            But avoid



            • Asking for help, clarification, or responding to other answers.

            • Making statements based on opinion; back them up with references or personal experience.


            To learn more, see our tips on writing great answers.




            draft saved


            draft discarded














            StackExchange.ready(
            function () {
            StackExchange.openid.initPostLogin('.new-post-login', 'https%3a%2f%2fmath.stackexchange.com%2fquestions%2f3011293%2flet-a-b-c-gt1-be-integers-such-that-gcda-%25e2%2588%2592-1-b-%25e2%2588%2592-1-c-%25e2%2588%2592-1-gt1-pro%23new-answer', 'question_page');
            }
            );

            Post as a guest















            Required, but never shown





















































            Required, but never shown














            Required, but never shown












            Required, but never shown







            Required, but never shown

































            Required, but never shown














            Required, but never shown












            Required, but never shown







            Required, but never shown







            Popular posts from this blog

            To store a contact into the json file from server.js file using a class in NodeJS

            Redirect URL with Chrome Remote Debugging Android Devices

            Dieringhausen